Compact subset of space of Continuous functions












5














Let $mathscr{C}[0,1]$ denote the set of continuous functions with bounded supremum and let $K={finmathscr{C}[0,1]|int_0^1f(t)dt=1}$. Then is $K$ compact in the space $mathscr{C}[0,1]$? Typically how do we characterize the compact spaces in the space of continuous functions? Will Heine-Borel property work here?



I think Heine-Borel would work, as $[0,1]$ is a compact Hausdorff space. Then, by using a function similar to spikes, or, somewhat like Dirac-Delta function, I think the space $K$ is not compact. Is my argument true? Any hints? Thanks beforehand.










share|cite|improve this question






















  • Have you heard of Arzela-Ascoli theorem?
    – Thomas Shelby
    2 days ago












  • Your heuristics is correct, but you still need a rigorous proof. You should produce an explicit example of a sequence in $K$ that has no limit points. Your idea will work.
    – Giuseppe Negro
    2 days ago










  • @ThomasShelby oh! I just saw the statement. It states that a subspace of continuous functions is compact iff the space is bounded and equicontinuous. In this case, the space is neither bounded nor equicontinuous, am I right
    – vidyarthi
    2 days ago








  • 1




    @ThomasShelby $K$ is definitely not a bounded subset of $C[0,1]$, see Fred's answer.
    – daw
    2 days ago






  • 2




    @ThomasShelby You're talking about different things here. Every element of $K$ is a bounded functions, as you said. But $K$ is not a bounded subset of the normed space $C[0,1]$. That's just two different meanings of boundedness (I guess the latter is exactly what you call uniformly bounded).
    – MaoWao
    2 days ago
















5














Let $mathscr{C}[0,1]$ denote the set of continuous functions with bounded supremum and let $K={finmathscr{C}[0,1]|int_0^1f(t)dt=1}$. Then is $K$ compact in the space $mathscr{C}[0,1]$? Typically how do we characterize the compact spaces in the space of continuous functions? Will Heine-Borel property work here?



I think Heine-Borel would work, as $[0,1]$ is a compact Hausdorff space. Then, by using a function similar to spikes, or, somewhat like Dirac-Delta function, I think the space $K$ is not compact. Is my argument true? Any hints? Thanks beforehand.










share|cite|improve this question






















  • Have you heard of Arzela-Ascoli theorem?
    – Thomas Shelby
    2 days ago












  • Your heuristics is correct, but you still need a rigorous proof. You should produce an explicit example of a sequence in $K$ that has no limit points. Your idea will work.
    – Giuseppe Negro
    2 days ago










  • @ThomasShelby oh! I just saw the statement. It states that a subspace of continuous functions is compact iff the space is bounded and equicontinuous. In this case, the space is neither bounded nor equicontinuous, am I right
    – vidyarthi
    2 days ago








  • 1




    @ThomasShelby $K$ is definitely not a bounded subset of $C[0,1]$, see Fred's answer.
    – daw
    2 days ago






  • 2




    @ThomasShelby You're talking about different things here. Every element of $K$ is a bounded functions, as you said. But $K$ is not a bounded subset of the normed space $C[0,1]$. That's just two different meanings of boundedness (I guess the latter is exactly what you call uniformly bounded).
    – MaoWao
    2 days ago














5












5








5


2





Let $mathscr{C}[0,1]$ denote the set of continuous functions with bounded supremum and let $K={finmathscr{C}[0,1]|int_0^1f(t)dt=1}$. Then is $K$ compact in the space $mathscr{C}[0,1]$? Typically how do we characterize the compact spaces in the space of continuous functions? Will Heine-Borel property work here?



I think Heine-Borel would work, as $[0,1]$ is a compact Hausdorff space. Then, by using a function similar to spikes, or, somewhat like Dirac-Delta function, I think the space $K$ is not compact. Is my argument true? Any hints? Thanks beforehand.










share|cite|improve this question













Let $mathscr{C}[0,1]$ denote the set of continuous functions with bounded supremum and let $K={finmathscr{C}[0,1]|int_0^1f(t)dt=1}$. Then is $K$ compact in the space $mathscr{C}[0,1]$? Typically how do we characterize the compact spaces in the space of continuous functions? Will Heine-Borel property work here?



I think Heine-Borel would work, as $[0,1]$ is a compact Hausdorff space. Then, by using a function similar to spikes, or, somewhat like Dirac-Delta function, I think the space $K$ is not compact. Is my argument true? Any hints? Thanks beforehand.







real-analysis general-topology functional-analysis






share|cite|improve this question













share|cite|improve this question











share|cite|improve this question




share|cite|improve this question










asked 2 days ago









vidyarthividyarthi

2,9341832




2,9341832












  • Have you heard of Arzela-Ascoli theorem?
    – Thomas Shelby
    2 days ago












  • Your heuristics is correct, but you still need a rigorous proof. You should produce an explicit example of a sequence in $K$ that has no limit points. Your idea will work.
    – Giuseppe Negro
    2 days ago










  • @ThomasShelby oh! I just saw the statement. It states that a subspace of continuous functions is compact iff the space is bounded and equicontinuous. In this case, the space is neither bounded nor equicontinuous, am I right
    – vidyarthi
    2 days ago








  • 1




    @ThomasShelby $K$ is definitely not a bounded subset of $C[0,1]$, see Fred's answer.
    – daw
    2 days ago






  • 2




    @ThomasShelby You're talking about different things here. Every element of $K$ is a bounded functions, as you said. But $K$ is not a bounded subset of the normed space $C[0,1]$. That's just two different meanings of boundedness (I guess the latter is exactly what you call uniformly bounded).
    – MaoWao
    2 days ago


















  • Have you heard of Arzela-Ascoli theorem?
    – Thomas Shelby
    2 days ago












  • Your heuristics is correct, but you still need a rigorous proof. You should produce an explicit example of a sequence in $K$ that has no limit points. Your idea will work.
    – Giuseppe Negro
    2 days ago










  • @ThomasShelby oh! I just saw the statement. It states that a subspace of continuous functions is compact iff the space is bounded and equicontinuous. In this case, the space is neither bounded nor equicontinuous, am I right
    – vidyarthi
    2 days ago








  • 1




    @ThomasShelby $K$ is definitely not a bounded subset of $C[0,1]$, see Fred's answer.
    – daw
    2 days ago






  • 2




    @ThomasShelby You're talking about different things here. Every element of $K$ is a bounded functions, as you said. But $K$ is not a bounded subset of the normed space $C[0,1]$. That's just two different meanings of boundedness (I guess the latter is exactly what you call uniformly bounded).
    – MaoWao
    2 days ago
















Have you heard of Arzela-Ascoli theorem?
– Thomas Shelby
2 days ago






Have you heard of Arzela-Ascoli theorem?
– Thomas Shelby
2 days ago














Your heuristics is correct, but you still need a rigorous proof. You should produce an explicit example of a sequence in $K$ that has no limit points. Your idea will work.
– Giuseppe Negro
2 days ago




Your heuristics is correct, but you still need a rigorous proof. You should produce an explicit example of a sequence in $K$ that has no limit points. Your idea will work.
– Giuseppe Negro
2 days ago












@ThomasShelby oh! I just saw the statement. It states that a subspace of continuous functions is compact iff the space is bounded and equicontinuous. In this case, the space is neither bounded nor equicontinuous, am I right
– vidyarthi
2 days ago






@ThomasShelby oh! I just saw the statement. It states that a subspace of continuous functions is compact iff the space is bounded and equicontinuous. In this case, the space is neither bounded nor equicontinuous, am I right
– vidyarthi
2 days ago






1




1




@ThomasShelby $K$ is definitely not a bounded subset of $C[0,1]$, see Fred's answer.
– daw
2 days ago




@ThomasShelby $K$ is definitely not a bounded subset of $C[0,1]$, see Fred's answer.
– daw
2 days ago




2




2




@ThomasShelby You're talking about different things here. Every element of $K$ is a bounded functions, as you said. But $K$ is not a bounded subset of the normed space $C[0,1]$. That's just two different meanings of boundedness (I guess the latter is exactly what you call uniformly bounded).
– MaoWao
2 days ago




@ThomasShelby You're talking about different things here. Every element of $K$ is a bounded functions, as you said. But $K$ is not a bounded subset of the normed space $C[0,1]$. That's just two different meanings of boundedness (I guess the latter is exactly what you call uniformly bounded).
– MaoWao
2 days ago










2 Answers
2






active

oldest

votes


















2














Consider the sequence $(f_n)_n$ given by $f_n(x) = 1+nsin(2pi x)$.
We have $(f_n)_n subseteq K$ but $$|f_n|_infty ge f_nleft(frac14right) = 1+nsinleft(fracpi2right) = 1+n $$



Hence $K$ isn't bounded so it cannot be compact.






An alternative argument: define a linear functional $phi : C[0,1] to mathbb{R}$ as $phi(f) = int_0^{1/2}f(t),dt$. We have that $phi$ is bounded and hence continuous with respect to the supremum norm.

If $K$ were compact, $phi|_K$ would be a bounded function. However, for the functions $(f_n)_n$ above we have
$$phi(f_n) = int_0^{1/2}f_n(t),dt = frac12 + frac{n}pi$$
which is a contradiction.






share|cite|improve this answer





























    6














    Let $f_n(t)=(n+1)t^n$. Then $f_n in K$ for all $n$. Can you proceed ?






    share|cite|improve this answer





















    • The sequence $f_n$ is not uniformly convergent( in fact , it diverges at $t=1$). So, does this imply that the limit $lim_{ntoinfty}int_0^1f_n(t)dt=0$ which is not in $K$. Is this the right way?
      – vidyarthi
      2 days ago








    • 2




      @vidyarthi No, just note that $|f_n|_{text{sup}} = n+1$ so that $K$ is unbounded in the norm so cannot be compact.
      – Henno Brandsma
      2 days ago











    Your Answer





    StackExchange.ifUsing("editor", function () {
    return StackExchange.using("mathjaxEditing", function () {
    StackExchange.MarkdownEditor.creationCallbacks.add(function (editor, postfix) {
    StackExchange.mathjaxEditing.prepareWmdForMathJax(editor, postfix, [["$", "$"], ["\\(","\\)"]]);
    });
    });
    }, "mathjax-editing");

    StackExchange.ready(function() {
    var channelOptions = {
    tags: "".split(" "),
    id: "69"
    };
    initTagRenderer("".split(" "), "".split(" "), channelOptions);

    StackExchange.using("externalEditor", function() {
    // Have to fire editor after snippets, if snippets enabled
    if (StackExchange.settings.snippets.snippetsEnabled) {
    StackExchange.using("snippets", function() {
    createEditor();
    });
    }
    else {
    createEditor();
    }
    });

    function createEditor() {
    StackExchange.prepareEditor({
    heartbeatType: 'answer',
    autoActivateHeartbeat: false,
    convertImagesToLinks: true,
    noModals: true,
    showLowRepImageUploadWarning: true,
    reputationToPostImages: 10,
    bindNavPrevention: true,
    postfix: "",
    imageUploader: {
    brandingHtml: "Powered by u003ca class="icon-imgur-white" href="https://imgur.com/"u003eu003c/au003e",
    contentPolicyHtml: "User contributions licensed under u003ca href="https://creativecommons.org/licenses/by-sa/3.0/"u003ecc by-sa 3.0 with attribution requiredu003c/au003e u003ca href="https://stackoverflow.com/legal/content-policy"u003e(content policy)u003c/au003e",
    allowUrls: true
    },
    noCode: true, onDemand: true,
    discardSelector: ".discard-answer"
    ,immediatelyShowMarkdownHelp:true
    });


    }
    });














    draft saved

    draft discarded


















    StackExchange.ready(
    function () {
    StackExchange.openid.initPostLogin('.new-post-login', 'https%3a%2f%2fmath.stackexchange.com%2fquestions%2f3066059%2fcompact-subset-of-space-of-continuous-functions%23new-answer', 'question_page');
    }
    );

    Post as a guest















    Required, but never shown

























    2 Answers
    2






    active

    oldest

    votes








    2 Answers
    2






    active

    oldest

    votes









    active

    oldest

    votes






    active

    oldest

    votes









    2














    Consider the sequence $(f_n)_n$ given by $f_n(x) = 1+nsin(2pi x)$.
    We have $(f_n)_n subseteq K$ but $$|f_n|_infty ge f_nleft(frac14right) = 1+nsinleft(fracpi2right) = 1+n $$



    Hence $K$ isn't bounded so it cannot be compact.






    An alternative argument: define a linear functional $phi : C[0,1] to mathbb{R}$ as $phi(f) = int_0^{1/2}f(t),dt$. We have that $phi$ is bounded and hence continuous with respect to the supremum norm.

    If $K$ were compact, $phi|_K$ would be a bounded function. However, for the functions $(f_n)_n$ above we have
    $$phi(f_n) = int_0^{1/2}f_n(t),dt = frac12 + frac{n}pi$$
    which is a contradiction.






    share|cite|improve this answer


























      2














      Consider the sequence $(f_n)_n$ given by $f_n(x) = 1+nsin(2pi x)$.
      We have $(f_n)_n subseteq K$ but $$|f_n|_infty ge f_nleft(frac14right) = 1+nsinleft(fracpi2right) = 1+n $$



      Hence $K$ isn't bounded so it cannot be compact.






      An alternative argument: define a linear functional $phi : C[0,1] to mathbb{R}$ as $phi(f) = int_0^{1/2}f(t),dt$. We have that $phi$ is bounded and hence continuous with respect to the supremum norm.

      If $K$ were compact, $phi|_K$ would be a bounded function. However, for the functions $(f_n)_n$ above we have
      $$phi(f_n) = int_0^{1/2}f_n(t),dt = frac12 + frac{n}pi$$
      which is a contradiction.






      share|cite|improve this answer
























        2












        2








        2






        Consider the sequence $(f_n)_n$ given by $f_n(x) = 1+nsin(2pi x)$.
        We have $(f_n)_n subseteq K$ but $$|f_n|_infty ge f_nleft(frac14right) = 1+nsinleft(fracpi2right) = 1+n $$



        Hence $K$ isn't bounded so it cannot be compact.






        An alternative argument: define a linear functional $phi : C[0,1] to mathbb{R}$ as $phi(f) = int_0^{1/2}f(t),dt$. We have that $phi$ is bounded and hence continuous with respect to the supremum norm.

        If $K$ were compact, $phi|_K$ would be a bounded function. However, for the functions $(f_n)_n$ above we have
        $$phi(f_n) = int_0^{1/2}f_n(t),dt = frac12 + frac{n}pi$$
        which is a contradiction.






        share|cite|improve this answer












        Consider the sequence $(f_n)_n$ given by $f_n(x) = 1+nsin(2pi x)$.
        We have $(f_n)_n subseteq K$ but $$|f_n|_infty ge f_nleft(frac14right) = 1+nsinleft(fracpi2right) = 1+n $$



        Hence $K$ isn't bounded so it cannot be compact.






        An alternative argument: define a linear functional $phi : C[0,1] to mathbb{R}$ as $phi(f) = int_0^{1/2}f(t),dt$. We have that $phi$ is bounded and hence continuous with respect to the supremum norm.

        If $K$ were compact, $phi|_K$ would be a bounded function. However, for the functions $(f_n)_n$ above we have
        $$phi(f_n) = int_0^{1/2}f_n(t),dt = frac12 + frac{n}pi$$
        which is a contradiction.







        share|cite|improve this answer












        share|cite|improve this answer



        share|cite|improve this answer










        answered 2 days ago









        mechanodroidmechanodroid

        27k62446




        27k62446























            6














            Let $f_n(t)=(n+1)t^n$. Then $f_n in K$ for all $n$. Can you proceed ?






            share|cite|improve this answer





















            • The sequence $f_n$ is not uniformly convergent( in fact , it diverges at $t=1$). So, does this imply that the limit $lim_{ntoinfty}int_0^1f_n(t)dt=0$ which is not in $K$. Is this the right way?
              – vidyarthi
              2 days ago








            • 2




              @vidyarthi No, just note that $|f_n|_{text{sup}} = n+1$ so that $K$ is unbounded in the norm so cannot be compact.
              – Henno Brandsma
              2 days ago
















            6














            Let $f_n(t)=(n+1)t^n$. Then $f_n in K$ for all $n$. Can you proceed ?






            share|cite|improve this answer





















            • The sequence $f_n$ is not uniformly convergent( in fact , it diverges at $t=1$). So, does this imply that the limit $lim_{ntoinfty}int_0^1f_n(t)dt=0$ which is not in $K$. Is this the right way?
              – vidyarthi
              2 days ago








            • 2




              @vidyarthi No, just note that $|f_n|_{text{sup}} = n+1$ so that $K$ is unbounded in the norm so cannot be compact.
              – Henno Brandsma
              2 days ago














            6












            6








            6






            Let $f_n(t)=(n+1)t^n$. Then $f_n in K$ for all $n$. Can you proceed ?






            share|cite|improve this answer












            Let $f_n(t)=(n+1)t^n$. Then $f_n in K$ for all $n$. Can you proceed ?







            share|cite|improve this answer












            share|cite|improve this answer



            share|cite|improve this answer










            answered 2 days ago









            FredFred

            44.4k1845




            44.4k1845












            • The sequence $f_n$ is not uniformly convergent( in fact , it diverges at $t=1$). So, does this imply that the limit $lim_{ntoinfty}int_0^1f_n(t)dt=0$ which is not in $K$. Is this the right way?
              – vidyarthi
              2 days ago








            • 2




              @vidyarthi No, just note that $|f_n|_{text{sup}} = n+1$ so that $K$ is unbounded in the norm so cannot be compact.
              – Henno Brandsma
              2 days ago


















            • The sequence $f_n$ is not uniformly convergent( in fact , it diverges at $t=1$). So, does this imply that the limit $lim_{ntoinfty}int_0^1f_n(t)dt=0$ which is not in $K$. Is this the right way?
              – vidyarthi
              2 days ago








            • 2




              @vidyarthi No, just note that $|f_n|_{text{sup}} = n+1$ so that $K$ is unbounded in the norm so cannot be compact.
              – Henno Brandsma
              2 days ago
















            The sequence $f_n$ is not uniformly convergent( in fact , it diverges at $t=1$). So, does this imply that the limit $lim_{ntoinfty}int_0^1f_n(t)dt=0$ which is not in $K$. Is this the right way?
            – vidyarthi
            2 days ago






            The sequence $f_n$ is not uniformly convergent( in fact , it diverges at $t=1$). So, does this imply that the limit $lim_{ntoinfty}int_0^1f_n(t)dt=0$ which is not in $K$. Is this the right way?
            – vidyarthi
            2 days ago






            2




            2




            @vidyarthi No, just note that $|f_n|_{text{sup}} = n+1$ so that $K$ is unbounded in the norm so cannot be compact.
            – Henno Brandsma
            2 days ago




            @vidyarthi No, just note that $|f_n|_{text{sup}} = n+1$ so that $K$ is unbounded in the norm so cannot be compact.
            – Henno Brandsma
            2 days ago


















            draft saved

            draft discarded




















































            Thanks for contributing an answer to Mathematics Stack Exchange!


            • Please be sure to answer the question. Provide details and share your research!

            But avoid



            • Asking for help, clarification, or responding to other answers.

            • Making statements based on opinion; back them up with references or personal experience.


            Use MathJax to format equations. MathJax reference.


            To learn more, see our tips on writing great answers.




            draft saved


            draft discarded














            StackExchange.ready(
            function () {
            StackExchange.openid.initPostLogin('.new-post-login', 'https%3a%2f%2fmath.stackexchange.com%2fquestions%2f3066059%2fcompact-subset-of-space-of-continuous-functions%23new-answer', 'question_page');
            }
            );

            Post as a guest















            Required, but never shown





















































            Required, but never shown














            Required, but never shown












            Required, but never shown







            Required, but never shown

































            Required, but never shown














            Required, but never shown












            Required, but never shown







            Required, but never shown







            Popular posts from this blog

            An IMO inspired problem

            Management

            Has there ever been an instance of an active nuclear power plant within or near a war zone?